Search found 234 matches


Good Question, I chose C first, but A makes sense.

by Java_85

Sun Nov 17, 2013 6:29 pm
Forum: Critical Reasoning
Topic: Volunteer Work
Replies: 5
Views: 4419
by Java_85

Sun Nov 17, 2013 5:56 pm
Forum: Sentence Correction
Topic: American revolution
Replies: 6
Views: 1830

An obstacle

The 151 member governments of the World Bank are expected to increase the bank's funding by $75 billion, though some United States legislators cite an obstacle to congressional passage being the concern that the bank's loans will help foreign producers compete with American businesses. (A) an obstac...

by Java_85

Sun Nov 17, 2013 5:40 pm
Forum: Sentence Correction
Topic: An obstacle
Replies: 9
Views: 1947

restaurant company

Although the restaurant company has recently added many new restaurants across the country and its sales have increased dramatically, its sales at restaurants open for more than a year have declined. A.the restaurant company has recently added many new restaurants across the country and its sales ha...

by Java_85

Sun Nov 17, 2013 5:29 pm
Forum: Sentence Correction
Topic: restaurant company
Replies: 5
Views: 1764

+1 for c, Other choices are out.

by Java_85

Sat Nov 16, 2013 2:13 pm
Forum: Critical Reasoning
Topic: editorial cannot be a good
Replies: 16
Views: 11360
by Java_85

Fri Nov 15, 2013 2:00 pm
Forum: Critical Reasoning
Topic: Aristotle LSAT CR - Q 14
Replies: 30
Views: 16620

Good Question, 9 it is.

by Java_85

Mon Nov 04, 2013 9:37 am
Forum: Problem Solving
Topic: digit issue
Replies: 26
Views: 13251
by Java_85

Sun Nov 03, 2013 8:26 am
Forum: Problem Solving
Topic: Mechanics Question
Replies: 25
Views: 12191
by Java_85

Fri Nov 01, 2013 2:06 pm
Forum: Data Sufficiency
Topic: Inequality
Replies: 30
Views: 13314

IMO B, A alone is not sufficient but B alone is.

by Java_85

Tue Oct 29, 2013 8:52 am
Forum: Data Sufficiency
Topic: Average value > Median
Replies: 24
Views: 11239

Good Question, A

by Java_85

Tue Oct 29, 2013 8:34 am
Forum: Critical Reasoning
Topic: Dolphins
Replies: 26
Views: 16261

B, other choices can't be inferred from the passage.

by Java_85

Mon Oct 28, 2013 7:36 am
Forum: Critical Reasoning
Topic: BTG question pool CR
Replies: 18
Views: 11553

Thanks Rahul, I still didn't get it why E is wrong, what is "subjunctive rule"?

by Java_85

Sat Oct 26, 2013 10:17 am
Forum: Sentence Correction
Topic: IRS provision
Replies: 5
Views: 1928
by Java_85

Sat Oct 26, 2013 10:04 am
Forum: Data Sufficiency
Topic: equations DS
Replies: 30
Views: 13103